GATE 2017 SET-2 CS Computer Science and information technology

[Q1 – Q18 Multiple choice Question(MCQ),carry ONE mark each (for each wrong answer: – 1/3) ]

Q1➡ | Compiler Design
Which of the following statements about parser is/are CORRECT?
I. Canonical LR is more powerful than SLR.
II. SLR is more powerful than LALR.
III. SLR is more powerful than Canonical LR.
i ➥ II only
ii ➥ I only
iii ➥ II and III only
iv ➥ III only

Show Answer With Best Explanation

Answer: II
Explanation: Upload Soon

More DiscussionExplanation On YouTubeParsers Help-Line

Q2➡ | Compiler Design
Match the following according to input (from the left column) to the compiler phase (in the right column) that processes it:
i ➥ P→(ii), Q→(i), R→(iii), S→(iv)
ii ➥ P→(iii), Q→(iv), R→(i), S→(ii)
iii ➥ P→(i), Q→(iv), R→(ii), S→(iii)
iv ➥ P→(ii), Q→(iii), R→(iv), S→(i)

Show Answer With Best Explanation

Answer: II
Explanation: Upload Soon

More DiscussionExplanation On YouTube Compiler phase Help-Line

Q3➡ | Theory of Computation
Let L1, L2 be any two context-free languages and R be any regular language. Then which of the following is/are CORRECT?
I. L1 U L2 is context Free.
II.
III. L1 – R is context Free.
IV.
i ➥ I only
ii ➥ I, II and IV only
iii ➥ II and IV only
iv ➥ I and III only

Show Answer With Best Explanation

Answer: IV
Explanation: Upload Soon

More DiscussionExplanation On YouTube Context Free language Help-Line

Q4➡ | Algorithms
Match the algorithms with their time complexities:
i ➥ P→(iv), Q→(iii), R→(i), S→(ii)
ii ➥ P→(iv), Q→(iii), R→(ii), S→(i)
iii ➥ P→(iii), Q→(iv), R→(ii), S→(i)
iv ➥ P→(iii), Q→(iv), R→(i), S→(ii)

Show Answer With Best Explanation

Answer: III
Explanation: Upload Soon

More DiscussionExplanation On YouTubeTime Complexity Help-Line

Q5➡ | Programming
Match the following:
i ➥ P→(iii), Q→(iv), R→(i), S→(ii)
ii ➥ P→(ii), Q→(iv), R→(iii), S→(i)
iii ➥ P→(ii), Q→(i), R→(iv), S→(iii)
iv ➥ P→(ii), Q→(iv), R→(i), S→(iii)

Show Answer With Best Explanation

Answer: III
Explanation: Upload Soon

More DiscussionExplanation On YouTubeProgramming concept Help-Line

Q6➡ | Digital Logic Design
The representation of the value of a 16-bit unsigned integer X in hexadecimal number system is BCA9. The representation of the value of X in octal number system is:
i ➥ 136252
ii ➥ 571247
iii ➥ 736251
iv ➥ 136251

Show Answer With Best Explanation

Answer: IV
Explanation: Upload Soon

More DiscussionExplanation On YouTube Number System Help-Line

Q7➡ | Digital Logic Design
Given the following binary number in 32-bit (single precision) IEEE-754 format:
00111110011011010000000000000000
The decimal value closest to this floating-point number is
i ➥ 2.27 × 10-1
ii ➥ 1.45 × 101
iii ➥ 1.45 × 10-1
iv ➥ 2.27 × 101

Show Answer With Best Explanation

Answer: I
Explanation: Upload Soon

More DiscussionExplanation On YouTube Number System Help-Line

Q8➡ | Engineering Mathematics
Let p, q, r denote the statements “It is raining”, “It is cold”, and “It is pleasant”, respectively. Then the statement “It is not raining and it is pleasant, and it is not pleasant only if it is raining and it is cold” is represented by
i ➥ (¬p ∧ r) ∨ (r → (p ∧ q))
ii ➥ (¬p ∧ r) ∧ ((p ∧ q) → ¬r)
iii ➥ (¬p ∧ r) ∨ ((p ∧ q) → ¬r)
iv ➥ (¬p ∧ r) ∧ (¬r → (p ∧ q))

Show Answer With Best Explanation

Answer: IV
Explanation: Upload Soon

More DiscussionExplanation On YouTube Propositional Logic Help-Line

Q9➡ | Engineering Mathematics
If f(x) = R sin(πx/2) + S, f'(1/2) = √2 and , then the constants R and S are, respectively.
i ➥
ii ➥
iii ➥
iv ➥

Show Answer With Best Explanation

Answer: III
Explanation: Upload Soon

More DiscussionExplanation On YouTube Calculus Help-Line

Q10➡ | Computer Network
Consider the following statements about the routing protocols, Routing Information Protocol (RIP) and Open Shortest Path First (OSPF) in an IPv4 network.
• I: RIP uses distance vector routing
• II: RIP packets are sent using UDP
• III: OSPF packets are sent using TCP
• IV: OSPF operation is based on link-state routing
Which of the statements above are CORRECT?
i ➥ I, II and III only
ii ➥ II, III and IV only
iii ➥ I and IV only
iv ➥ I, II and IV only

Show Answer With Best Explanation

Answer: IV
Explanation: Upload Soon

More Discussion Explanation On YouTube Routing Protocol Help-Line

Q11➡ | Operating Systems
In a file allocation system, which of the following allocation scheme(s) can be used if no external fragmentation is allowed?
I. Contiguous
II. Linked
III. Indexed
i ➥ II and III only
ii ➥ I and III only
iii ➥ II only
iv ➥ III only

Show Answer With Best Explanation

Answer: I
Explanation: Upload Soon

More DiscussionExplanation On YouTubeFile Allocation MethodsHelp-Line

Q12➡ | Operating Systems
Which of the following is/are shared by all the threads in a process?
I. Program counter
II. Stack
III. Address space
IV. Registers
i ➥ IV only
ii ➥ III and IV only
iii ➥ III only
iv ➥ I and II only

Show Answer With Best Explanation

Answer: III
Explanation: Upload Soon

More DiscussionExplanation On YouTubeThreads Help-Line

Q13➡ | Computer Network
Consider socket API on a Linux machine that supports connected UDP sockets. A connected UDP socket is a UDP socket on which connect function has already been called. Which of the following statement is/are CORRECT?
I. A connected UDP socket can be used to communicate with multiple peers simultaneously.
II. A process can successfully call connect function again for an already connected UDP socket.
i ➥ Both I and II
ii ➥ Neither I nor II
iii ➥ II only
iv ➥ I only

Show Answer With Best Explanation

Answer: III
Explanation: Upload Soon

More DiscussionExplanation On YouTubeUDP Help-Line

Q14➡ | Database Management System
An ER model of a database consists of entity types A and B. These are connected by a relationship R which does not have its own attribute. Under which one of the following conditions, can the relational table for R be merged with that of A?
i ➥ Relationship R is one-to-many and the participation of A in R is partial.
ii ➥ Relationship R is many-to-one and the participation of A in R is total.
iii ➥ Relationship R is one-to-many and the participation of A in R is partial.
iv ➥ Relationship R is one-to-many and the participation of A in R is total.

Show Answer With Best Explanation

Answer: II
Explanation: Upload Soon

More DiscussionExplanation On YouTubeER Model Help-Line

Q15➡ | Theory of Computation
Identify the language generated by the following grammar, where S is the start variable.
S → XY
X → aX|a
Y → aYb|ϵ
i ➥ {am bn │ m ≥ n, n > 0}
ii ➥ {am bn │ m ≥ n, n ≥ 0}
iii ➥ {am bn │ m > n, n ≥ 0}
iv ➥ {am bn │ m > n, n > 0}

Show Answer With Best Explanation

Answer: III
Explanation: Upload Soon

More DiscussionExplanation On YouTubeFinite Automata Help-Line

Q16➡ | Data Structure
The Breadth First Search (BFS) algorithm has been implemented using the queue data structure. Which one of the following is a possible order of visiting the nodes in the graph below?
 The Breadth First Search (BFS) algorithm has been implemented using the queue data structure. Which one of the following is a possible order of visiting the nodes in the graph below?
i ➥ NQMPOR
ii ➥ MNOPQR
iii ➥ QMNROP
iv ➥ POQNMR

Show Answer With Best Explanation

Answer: IV
Explanation: Upload Soon

More DiscussionExplanation On YouTubeNetwork Security Help-Line

Q17➡ | Programming
Consider the following function implemented in C:

The output of invoking printxy(1, 1) is
i ➥ 1, 1
ii ➥ 1, 0
iii ➥ 0, 1
iv ➥ 0, 0

Show Answer With Best Explanation

Answer: II
Explanation: Upload Soon

More DiscussionExplanation On YouTubePointer Help-Line

Q18➡ | Data Structure
A circular queue has been implemented using a singly linked list where each node consists of a value and a single pointer pointing to the next node. We maintain exactly two external pointers FRONT and REAR pointing to the front node and the rear node of the queue, respectively. Which of the following statements is/are CORRECT for such a circular queue, so that insertion and deletion operations can be performed in O(1) time?
I: Next pointer of front node points to the rear node.
II: Next pointer of rear node points to the front node.
i ➥ Neither I nor II
ii ➥ Both I and II
iii ➥ II only
iv ➥ I only

Show Answer With Best Explanation

Answer: III
Explanation: Upload Soon

More DiscussionExplanation On YouTubeQueue and Linked List Help-Line

[Q19 – Q25 Multiple choice Question(MCQ),carry ONE mark each (no negative marks) ]

Q19➡ | Digital Logic Design
Consider a quadratic equation x2 – 13x + 36 = 0 with coefficients in a base b. The solutions of this equation in the same base b are x = 5 and x = 6. Then b=_________.

Show Answer With Best Explanation

Answer: 8
Explanation: Upload Soon

More DiscussionExplanation On YouTubeNumber System Help-Line

Q20➡ | Engineering Mathematics
G is an undirected graph with n vertices and 25 edges such that each vertex of G has degree at least 3. Then the maximum possible value of n is __________.

Show Answer With Best Explanation

Answer: 16
Explanation: Upload Soon

More DiscussionExplanation On YouTubeGraph Theory Help-Line

Q21➡ | Engineering Mathematics
Let P= and Q= be two matrices.
Then the rank of P+Q is __________.

Show Answer With Best Explanation

Answer: 2
Explanation: Upload Soon

More DiscussionExplanation On YouTube Matrices Help-Line

Q22➡ | Engineering Mathematics
Consider the set X = {a,b,c,d e} under the partial ordering
R = {(a,a),(a,b),(a,c),(a,d),(a,e),(b,b),(b,c),(b,e),(c,c),(c,e),(d,d),(d,e),(e,e)}.
The Hasse diagram of the partial order (X,R) is shown below:

The minimum number of ordered pairs that need to be added to R to make (X,R) a lattice is ___________.

Show Answer With Best Explanation

Answer: 0
Explanation: Upload Soon

More DiscussionExplanation On YouTubeSet Theory Help-Line

Q23➡ | Computer Network
The maximum number of IPv4 router addresses that can be listed in the record route (RR) option field of an IPv4 header is ________.

Show Answer With Best Explanation

Answer: 9
Explanation: Upload Soon

More DiscussionExplanation On YouTubeIPV4 Header Help-Line

Q24➡ | Database Management System
Consider the following tables T1 and T2.

In table T1, P is the primary key and Q is the foreign key referencing R in table T2 with on-delete cascade and on-update cascade. In table T2, R is the primary key and S is the foreign key referencing P in table T1 with on-delete set NULL and on-update cascade. In order to delete record 〈3,8〉 from table T1, the number of additional records that need to be deleted from table T1 is ______.

Show Answer With Best Explanation

Answer: 0
Explanation: Upload Soon

More DiscussionExplanation On YouTubeReferential Integrity Help-Line

Q25➡ | Theory of Computation
The minimum possible number of a deterministic finite automation that accepts the regular language L = {w1aw2 | w1, w2 ∈ {a,b}*, |w1| = 2,|w2| ≥ 3} is ___________.

Show Answer With Best Explanation

Answer: 8
Explanation: Upload Soon

More DiscussionExplanation On YouTubeDFA Help-Line

[Q26 – Q42 Multiple choice Question(MCQ),carry TWO mark each (for each wrong answer: – 2/3) ]

Q26➡ | Engineering Mathematics
For any discrete random variable X, with probability mass function P(X=j)=pj, pj≥0, j∈{0, …, N} and , define the polynomial function

. For a certain discrete random variable Y, there exists a scalar β∈[0,1] such that gy(Z)=(1-β+βz)N. The expectation of Y is
i ➥ Nβ(1 – β)
ii ➥
iii ➥ N(1 – β)
iv ➥ Not expressible in terms of N and β alone

Show Answer With Best Explanation

Answer: II
Explanation: Upload Soon

More DiscussionExplanation On YouTubeProbability Help-Line

Q27➡ | Algorithms
Consider the recurrence function

Then T(n) in terms of Θ notation is
i ➥ Θ(n)
ii ➥ Θ(√n)
iii ➥ Θ(log⁡n)
iv ➥ Θ(log⁡log⁡n)

Show Answer With Best Explanation

Answer: III
Explanation: Upload Soon

More DiscussionExplanation On YouTube Time Complexity Help-Line

Q28➡ | Computer Organization
In a two-level cache system, the access times of L1 and L2 caches are 1 and 8 clock cycles, respectively. The miss penalty from the L2 cache to main memory is 18 clock cycles. The miss rate of L1 cache is twice that of L2. The average memory access time (AMAT) of this cache system is 2 cycles. The miss rates of L1 and L2 respectively are:
i ➥ 0.111 and 0.056
ii ➥ 0.056 and 0.111
iii ➥ 0.0892 and 0.1784
iv ➥ 0.1784 and 0.0892

Show Answer With Best Explanation

Answer: I
Explanation: Upload Soon

More DiscussionExplanation On YouTubeCache Memory Help-Line

Q29➡ | Digital Logic Design
Given f(w,x,y,z) = Σm(0,1,2,3,7,8,10) + Σd(5,6,11,15), where d represents the don’t-care condition in Karnaugh maps. Which of the following is a minimum product-of-sums (POS) form of f(w,x,y,z)?
i ➥ f = (w’ + z’) (x’ + z)
ii ➥ f = (w’ + z) (x + z)
iii ➥ f = (w + z) (x’ + z)
iv ➥ f = (w + z’) (x’ + z)

Show Answer With Best Explanation

Answer: I
Explanation: Upload Soon

More DiscussionExplanation On YouTube Boolean Algebra Help-Line

Q30➡ | Digital Logic Design
If w, x, y, z are Boolean variables, then which one of the following is INCORRECT?
i ➥ wx + w(x + y) + x(x + y) = x + wy
ii ➥
iii ➥
iv ➥ (w + y)(wxy + wyz) = wxy + wyz

Show Answer With Best Explanation

Answer: III
Explanation: Upload Soon

More DiscussionExplanation On YouTube Boolean Algebra Help-Line

Q31➡ | Engineering Mathematics
P and Q are considering to apply for job. The probability that p applies for job is 1/4. The probability that P applies for job given that Q applies for the job 1/2 and The probability that Q applies for job given that P applies for the job 1/3.The probability that P does not apply for job given that Q does not apply for the job
i ➥ 4/5
ii ➥ 5/6
iii ➥ 7/8
iv ➥ 11/12

Show Answer With Best Explanation

Answer: I
Explanation: Upload Soon

More DiscussionExplanation On YouTubeProbabilityHelp-Line

Q32➡ | Programming
Consider the C program fragment below which is meant to divide x and y using repeated subtractions. The variables x, y, q and r are all unsigned int.

Which of the following conditions on the variables x, y, q and r before the execution of the fragment will ensure that the loop terminates in a state satisfying the condition x == (y*q + r)?
i ➥ (q == 0) && (y > 0)
ii ➥ (q == 0) && (r == x) && (y > 0)
iii ➥ (x > 0) && (r == x) && (y > 0)
iv ➥ (q == r) && (r == 0)

Show Answer With Best Explanation

Answer: II
Explanation: Upload Soon

More DiscussionExplanation On YouTubeProgram Help-Line

Q33➡ | Data Structure
The pre-order traversal of a binary search tree is given by 12, 8, 6, 2, 7, 9, 10, 16, 15, 19, 17, 20. Then the post-order traversal of this tree is:
i ➥ 7, 6, 2, 10, 9, 8, 15, 16, 17, 20, 19, 12
ii ➥ 7, 2, 6, 8, 9, 10, 20, 17, 19, 15, 16, 12
iii ➥ 2, 7, 6, 10, 9, 8, 15, 17, 20, 19, 16, 12
iv ➥ 2, 6, 7, 8, 9, 10, 12, 15, 16, 17, 19, 20

Show Answer With Best Explanation

Answer: III
Explanation: Upload Soon

More DiscussionExplanation On YouTubeBinary Trees Help-Line

Q34➡ | Computer Network
Consider two hosts X and Y, connected by a single direct link of rate 106bits/sec. The distance between the two hosts is 10,000 km and the propagation speed along the link is 2×108m/sec. Host X sends a file of 50,000 bytes as one large message to host Y continuously. Let the transmission and propagation delays be p milliseconds and q milliseconds, respectively. Then the values of p and q are
i ➥ p=400 and q=50
ii ➥ p=100 and q=50
iii ➥ p=50 and q=400
iv ➥ p=50 and q=100

Show Answer With Best Explanation

Answer: I
Explanation:
Given,
Data rate (or Bandwidth) = 106 bits/sec
Distance = 10,000 km
Speed = 2×108 m/sec
Data Length = 50,000 Bytes

Ask,
Transmission Delay = ?
Propagation Delay = ?

Formula,
Consider two hosts X and Y, connected by a single direct link of rate 106bits/sec. The distance between the two hosts is 10,000 km and the propagation speed along the link is 2×108m/sec. Host X sends a file of 50,000 bytes as one large message to host Y continuously. Let the transmission and propagation delays be p milliseconds and q milliseconds, respectively. Then the values of p and q are

Consider two hosts X and Y, connected by a single direct link of rate 106bits/sec. The distance between the two hosts is 10,000 km and the propagation speed along the link is 2×108m/sec. Host X sends a file of 50,000 bytes as one large message to host Y continuously. Let the transmission and propagation delays be p milliseconds and q milliseconds, respectively. Then the values of p and q are

Calculation,
Transmission Delay
Propagation Delay
So,Option(I) is correct.

More Discussion Explanation On YouTube Flow Control Help-Line

Q35➡ | Digital Logic Design
Consider a binary code that consists of only four valid code words as given below:
00000, 01011, 10101, 11110
Let the minimum Hamming distance of the code be p and the maximum number of erroneous bits that can be corrected by the code be q. Then the values of p and q are
i ➥ p=4 and q=2
ii ➥ p=4 and q=1
iii ➥ p=3 and q=2
iv ➥ p=3 and q=1

Show Answer With Best Explanation

Answer: IV
Explanation: Upload Soon

More DiscussionExplanation On YouTubeNumber System Help-Line

Q36➡ | Operating System
A system shares 9 tape drives. The current allocation and maximum requirement of tape drives for three processes are shown below:
A system shares 9 tape drives. The current allocation and maximum requirement of tape drives for three processes are shown below: Which of the following best describe current state of the system?
Which of the following best describe current state of the system?
i ➥ Not Safe, Not Deadlocked
ii ➥ Not Safe, Deadlocked
iii ➥ Safe, Not Deadlocked
iv ➥ Safe, Deadlocked

Show Answer With Best Explanation

Answer: III
Explanation: Upload Soon

More DiscussionExplanation On YouTubeDeadlock Help-Line

Q37➡ | Compiler Design
Consider the following expression grammar G:
E → E – T | T
T → T + F | F
F → (E) | id
Which of the following grammars is not left recursive, but is equivalent to G?
i ➥
ii ➥
iii ➥
iv ➥

Show Answer With Best Explanation

Answer: III
Explanation: Upload Soon

More DiscussionExplanation On YouTubeLeft Recursive GrammerHelp-Line

Q38➡ | Digital Logic Design
The next state table of a 2-bit saturating up-counter is given below.
The counter is built as a synchronous sequential circuit using T flip-flops. The expressions for T1 and T0 are
i ➥ T1=Q1Q0 , T0 = Q’1Q’0
ii ➥ T1=Q’1Q0 , T0 = Q’1 + Q’0
iii ➥ T1=Q1 + Q0 , T0 = Q’1 + Q’0
iv ➥ T1=Q’1Q0 , T0 = Q1 + Q0

Show Answer With Best Explanation

Answer: II
Explanation: Upload Soon

More DiscussionExplanation On YouTubeSequential Circuits Help-Line

Q39➡ | Theory of Computation
Let L(R) be the language represented by regular expression R. Let L(G) be the language generated by a context free grammar G. Let L(M) be the language accepted by a Turing machine M.

Which of the following decision problems are undecidable?
• I. Given a regular expression R and a string w, is w ∈ L(R)?
• II. Given a context-free grammar G, is L(G) = ∅?
• III. Given a context-free grammar G, is L(G) = Σ* for some alphabet Σ?
• IV. Given a Turing machine M and a string w, is w ∈ L(M)?
i ➥ III and IV only
ii ➥ II, III and IV only
iii ➥ II and III only
iv ➥ I and IV only

Show Answer With Best Explanation

Answer: I
Explanation: Upload Soon

More DiscussionExplanation On YouTubeDecidability and UndecidabilityHelp-Line

Q40➡ | Theory of Computation
Consider the following languages:
L1 = {ap│p is a prime number}
L2 = {an bmc2m | n ≥ 0, m ≥ 0}
L3 = {an bn c2n │ n ≥ 0} 
L4 = {an bn │ n ≥ 1}
Which of the following are CORRECT?
I. L1 is context-free but not regular.
II. L2 is not context-free.
III. L3 is not context-free but recursive. 
IV. L4 is deterministic context-free.
i ➥ III and IV only
ii ➥ I and IV only
iii ➥ II and III only
iv ➥ I, II and IV only

Show Answer With Best Explanation

Answer: I
Explanation: Upload Soon

More DiscussionExplanation On YouTubeContext Free LanguageHelp-Line

Q41➡ | Theory of Computation
Let δ denote the transition function and denote the extended transition function of the ε-NFA whose transition table is given below:

Then is
i ➥ {q0,q2,q3}
ii ➥ {q0,q1,q2}
iii ➥ {q0,q1,q3}
iv ➥

Show Answer With Best Explanation

Answer: II
Explanation: Upload Soon

More DiscussionExplanation On YouTubeNFA Help-Line

Q42➡ | Algorithms
Consider the following C function.

Time complexity of fun in terms of Θ notation is
i ➥ Θ(n2 logn)
ii ➥ Θ(n log⁡n)
iii ➥ Θ(n2)
iv ➥ Θ(n√n)

Show Answer With Best Explanation

Answer: II
Explanation: Upload Soon

More DiscussionExplanation On YouTubeTime Complexity Help-Line

[Q43 – Q55 Multiple choice Question(MCQ),carry TWO mark each (no negative marks) ]

Q43➡ | Engineering Mathematics
If a random variable X has a Poisson distribution with mean 5, then the expectation E[(X + 2)2] equals ________.

Show Answer With Best Explanation

Answer: 54
Explanation: Upload Soon

More DiscussionExplanation On YouTubeProbability Help-Line

Q44➡ | Engineering Mathematics
If the ordinary generating function of a sequence is , then a3 – a0 is equal to __________.

Show Answer With Best Explanation

Answer: 15
Explanation: Upload Soon

More DiscussionExplanation On YouTubeCombinatorics Help-Line

Q45➡ | Database Management System
Consider the following database table named top_scorer.

SELECT ta.player FROM top_scorer AS ta
WHERE ta.goals > ALL (SELECT tb.goals
FROM top_scorer AS tb
WHERE tb.country = ‘Spain’)

AND ta.goals > ANY (SELECT tc.goals
FROM top_scorer AS tc
WHERE tc.country = ‘Germany’)

The number of tuples returned by the above SQL query is _________.

Show Answer With Best Explanation

Answer: 7
Explanation: Upload Soon

More DiscussionExplanation On YouTubeSQL Help-Line

Q46➡ | Computer Organization
The read access times and the hit ratios for different caches in a memory hierarchy are as given below:

The read access time of main memory is 90 nanoseconds. Assume that the caches use the referred-word-first read policy and the write back policy. Assume that all the caches are direct mapped caches. Assume that the dirty bit is always 0 for all the blocks in the caches. In execution of a program, 60% of memory reads are for instruction fetch and 40% are for memory operand fetch. The average read access time in nanoseconds (up to 2 decimal places) is _________.

Show Answer With Best Explanation

Answer: 4.72
Explanation: Upload Soon

More DiscussionExplanation On YouTubeCache Memory Help-Line

Q47➡ | Database Management System
Two transactions T1 and T2 are given as
• T1: r1(X) w1(X) r1(Y) w1(Y)
• T2: r2(Y) w2(Y) r2(Z) w2(Z)
where ri(V) denotes a read operation by transaction Ti on a variable V and wi(V) denotes a write operation by transaction Ti on a variable V. The total number of conflict serializable schedules that can be formed by T1 and T2 is _______.

Show Answer With Best Explanation

Answer: 54
Explanation: Upload Soon

More DiscussionExplanation On YouTubeTransaction ManagementHelp-Line

Q48➡ | Programming
Consider the following snippet of a C program. Assume that swap(&x, &y) exchanges the contents of x and y.

The output of the program is _______.

Show Answer With Best Explanation

Answer: 3
Explanation: Upload Soon

More DiscussionExplanation On YouTubeProgram Help-Line

Q49➡ | Programming
Consider the following C program:

The output of the program is ________.

Show Answer With Best Explanation

Answer: 0
Explanation: Upload Soon

More DiscussionExplanation On YouTubeProgram Help-Line

Q50➡ | Computer Organization
Consider a machine with a byte addressable main memory of 232 bytes divided into blocks of size 32 bytes. Assume that a direct mapped cache having 512 cache lines is used with this machine. The size of the tag field in bits is _______.

Show Answer With Best Explanation

Answer: 18
Explanation: Upload Soon

More DiscussionExplanation On YouTubeCache Memory Help-Line

Q51➡ | Engineering Mathematics
If the characteristic polynomial of a 3 × 3 matrix M over ℝ (the set of real numbers) is λ3 – 4λ2 + aλ + 30, a ∈ ℝ, and one eigenvalue of M is 2, then the largest among the absolute values of the eigenvalues of M is __.

Show Answer With Best Explanation

Answer: 5
Explanation: Upload Soon

More DiscussionExplanation On YouTubeLinear Algebra Help-Line

Q52➡ | Operating Systems
Consider the set of processes with arrival time (in milliseconds), CPU burst time (in milliseconds), and priority (0 is the highest priority) shown below. None of the processes have I/O burst time.

The average waiting time (in milliseconds) of all the processes using preemptive priority scheduling algorithm is ________.

Show Answer With Best Explanation

Answer: 29
Explanation: Upload Soon

More DiscussionExplanation On YouTubeProcess Scheduling Help-Line

Q53➡ | Algorithms
A message is made up entirely of characters from the set X = {P, Q, R, S, T}. The table of probabilities for each of the characters is shown below:

If a message of 100 characters over X is encoded using Huffman coding, then the expected length of the encoded message in bits is _______.

Show Answer With Best Explanation

Answer: 225
Explanation: Upload Soon

More DiscussionExplanation On YouTubeHuffman Coding Help-Line

Q54➡ | Database Management System
In a B+ tree, if the search-key value is 8 bytes long, the block size is 512 bytes and the block pointer size is 2 bytes, then the maximum order of the B+ tree is ________.

Show Answer With Best Explanation

Answer: 52
Explanation: Upload Soon

More DiscussionExplanation On YouTubeFile Structure Help-Line

Q55➡ | Programming
Consider the following C program.

The output of the program is _________.

Show Answer With Best Explanation

Answer: 2
Explanation: Upload Soon

More DiscussionExplanation On YouTubeProgram Help-Line

error: Content is protected !!
Open chat
1
Hi,how Can We Help You ?